RegistrierenRegistrieren   LoginLogin   FAQFAQ    SuchenSuchen   
Konstante Metrik
 
Neue Frage »
Antworten »
    Foren-Übersicht -> Mechanik
Autor Nachricht
Triton
Gast





Beitrag Triton Verfasst am: 23. März 2021 22:36    Titel: Konstante Metrik Antworten mit Zitat

Meine Frage:

Grüße,

Ich sitze an einer Aufgabe für meinen Kurs zur ART. Dort ist zu zeigen:


""" """ """
Zeige: Wenn die Metrik nicht explizit von einer Koordinate (z.B. ) abhängt, dann ist konstant entlang jeder Geodäte.
""" """ """

Meine Ideen:
Meine Berechnungen hierzu sind wie folgt:

Die Lagrange Funktion, welche die Geodätengleichung



erfüllt, ist



Wenn nun die Metrik unabhängig ist, dann folgt nach den Euler-Lagrange Gleichungen:















Ich weiß nun leider nicht, wie der letzte Schritt zu rechtfertigen ist, d.h. wieso



sein soll. Weiß jemand weiter?


Grüße,

Triton


P.S.: Ich habe erst dieses Semester mit ART begonnen, kenne mich daher noch nicht so gut aus.
Ein Zwölftel
Gast





Beitrag Ein Zwölftel Verfasst am: 23. März 2021 22:41    Titel: Impuls und Kratstoß Antworten mit Zitat

Hey sorry kann dir leider nicht helfen, da ich noch auf dem Gymnasium bin.
Könntest du mir vielleicht bitte bei meiner (für dich bestimmt leichten) Aufgabe helfen.
Würde mich sehr freuen, weil ich da gerade ziemlich hänge.
index_razor



Anmeldungsdatum: 14.08.2014
Beiträge: 3259

Beitrag index_razor Verfasst am: 24. März 2021 08:08    Titel: Re: Wenn Metrik unabhänig von Koordinate, dann Metrik konsta Antworten mit Zitat

Triton hat Folgendes geschrieben:

Ich weiß nun leider nicht, wie der letzte Schritt zu rechtfertigen ist, d.h. wieso



sein soll. Weiß jemand weiter?


Laut deiner Rechnung scheint einfach die b-te Komponente des Vektors bzgl. der Basis zu bezeichnen. Natürlich muß dann gelten

Triton
Gast





Beitrag Triton Verfasst am: 24. März 2021 15:31    Titel: Antworten mit Zitat

Danke für deine Antwort! Ich habe nun ein anderes Argument gefunden, von dem ich glaube, dass es korrekt ist.


Die Metrik kann ich als Tensorprodukt zweier Dualvektoren darstellen:



Damit ist dann



und damit folgt die Behauptung.


MfG Triton
Neue Frage »
Antworten »
    Foren-Übersicht -> Mechanik